Understanding KKT Conditions for Minimization Problems with Constraints

Click For Summary
The discussion focuses on the application of the Karush-Kuhn-Tucker (KKT) conditions for a minimization problem with constraints. The KKT conditions are essential for determining optimal solutions, requiring that the objective function and constraints be stationary, dual and primal feasible, and satisfy complementary slackness. Two scenarios are considered for the product constraint equaling one: when all variables x_i are equal to one, resulting in a sum of n, and an uncertain second case where the product approaches one. The conversation seeks clarity on whether this approach to the problem is correct. Understanding these conditions is crucial for solving constrained optimization problems effectively.
hoffmann
Messages
65
Reaction score
0
what does it mean to write out the kkt conditions and find x* for the following problem:

minimize f(x) = \sum x_i subject to \prod x_i = 1 and x_i \geq 0 for 1<= i <= n. the bounds on the sum and product are from i = 1 to n.
 
Physics news on Phys.org
Well, what is the "kkt" (Karush–Kuhn–Tucker) theorem?
 
basically the kkt conditions need to be satisfied if the solution is optimal. you have the two constraints as your functions (say g and h) -- both these and the objective function need to be stationary, dual and primal feasible, and satisfy complementary slackness.

anyway, so i think there are two cases for the product to be equal to one: one is when all the x_i are equal to 1 and the other is when the product of the x_i's somehow approaches 1. in the first case, the sum would just give n since all the x_i's equal 1, and the second case...well I'm not so sure.

am i thinking about this problem in the right way?
 
Question: A clock's minute hand has length 4 and its hour hand has length 3. What is the distance between the tips at the moment when it is increasing most rapidly?(Putnam Exam Question) Answer: Making assumption that both the hands moves at constant angular velocities, the answer is ## \sqrt{7} .## But don't you think this assumption is somewhat doubtful and wrong?

Similar threads

  • · Replies 13 ·
Replies
13
Views
2K
Replies
2
Views
1K
Replies
3
Views
1K
Replies
10
Views
2K
Replies
0
Views
835
Replies
3
Views
2K
Replies
4
Views
2K
  • · Replies 2 ·
Replies
2
Views
1K
  • · Replies 7 ·
Replies
7
Views
1K
Replies
2
Views
2K